Q17

 
sunhwa2881
Thanks Received: 0
Forum Guests
 
Posts: 21
Joined: August 10th, 2010
 
 
 

Q17

by sunhwa2881 Mon Aug 23, 2010 8:54 pm

I was able to eliminate answer choices A, C, and D. However, i don't know why B can't be true. If 7, 8, and 9 are Y, G, and O, respectively, why is this wrong?
User avatar
 
ManhattanPrepLSAT2
Thanks Received: 311
Atticus Finch
Atticus Finch
 
Posts: 303
Joined: July 14th, 2009
 
 
 

Re: PT 20 S3 G3 A jeweler makes

by ManhattanPrepLSAT2 Tue Aug 24, 2010 7:18 pm

Sorry - what question are you talking about?
 
sunhwa2881
Thanks Received: 0
Forum Guests
 
Posts: 21
Joined: August 10th, 2010
 
 
 

Re: PT 20 S3 G3 A jeweler makes

by sunhwa2881 Tue Aug 24, 2010 7:53 pm

Sorry... It's Q 17. Thanks
User avatar
 
ManhattanPrepLSAT2
Thanks Received: 311
Atticus Finch
Atticus Finch
 
Posts: 303
Joined: July 14th, 2009
 
This post thanked 2 times.
 
 

Re: PT 20 S3 G3 Q 17 A jeweler makes

by ManhattanPrepLSAT2 Wed Aug 25, 2010 2:48 pm

Oh boy is this a killer problem -- one of the toughest I've ever seen.

(B) is incorrect because of the last constraint -- it's easy to read that constraint to mean "this 9 bead strand has to have all the colors" but that's not actually what it says.

What that last constraint means is that if we were to take any 8 bead part of the strand, we'd have to be able to get all 5 colors from that part. If we put y, g, and o into 7,8, and 9, then just look at the first 8 beads, we wouldn't have all 5 colors in those 8 beads.
 
opulence2001
Thanks Received: 4
Forum Guests
 
Posts: 43
Joined: November 10th, 2010
 
 
 

Re: PT 20 S3 G3 Q 17 A jeweler makes

by opulence2001 Mon Nov 15, 2010 8:24 pm

I'm still having some trouble with this one.
The question asks if you have PY_PY_ _ _ _ then what could be true.
I know the third must be R. But the rest are a toss up for me.
why couldnt you have PYRPYGOGR? The seventh bead is O, the eigth bead is green, and the ninth bead is read (answers A, B, and D respectively). From beads 1 to 8 each colour is represented, and I see no other violations.

For this one I chose D because I figured R in the ninth position will be in front of PY (positions 1 and 2) in the bracelet.
 
s0630506
Thanks Received: 0
Vinny Gambini
Vinny Gambini
 
Posts: 4
Joined: September 08th, 2010
 
 
 

Re: PT 20 S3 G3 Q 17 A jeweler makes

by s0630506 Mon Jan 10, 2011 8:59 pm

according to the 1st constraint if you have py you need r preceding and following it so 1-6 have to be PYRPYR leaving only 2 spots open for O and G, and since O cannot go next to R, G would have to go in the 7th space and O in the eighth to satisfy the final constraint of any portion of eight beads must include at least one color of each
 
giladedelman
Thanks Received: 833
LSAT Geek
 
Posts: 619
Joined: April 04th, 2010
 
 
 

Re: PT 20 S3 G3 Q 17 A jeweler makes

by giladedelman Wed Jan 12, 2011 11:58 pm

Good explanation!
 
zaidjawed
Thanks Received: 0
Forum Guests
 
Posts: 15
Joined: October 11th, 2012
 
 
 

Re: Q17

by zaidjawed Thu Oct 18, 2012 9:50 pm

Hi Guys,
I seem to be on the same page with sunhwa2881 here. He/she asks why B could not be true and I seem to be sharing the same train of though. After going through all the explanations posted after, I see why E is the right answer. However, why cant I have G go 8th and 0range go 9? If that were the case, B could also be true, no?
 
zaidjawed
Thanks Received: 0
Forum Guests
 
Posts: 15
Joined: October 11th, 2012
 
 
 

Re: Q17

by zaidjawed Thu Oct 18, 2012 9:51 pm

Oh!! Now I get it! The 8 consecutive beads must contain all colours. Hence G must go in 7th. Shrewd little question!

Thanks guys